LSAT and Law School Admissions Forum

Get expert LSAT preparation and law school admissions advice from PowerScore Test Preparation.

 Administrator
PowerScore Staff
  • PowerScore Staff
  • Posts: 8916
  • Joined: Feb 02, 2011
|
#22833
Complete Question Explanation

Must Be True. The correct answer choice is (C)

The author recommends that businesses introduce minor variations into their operating system software, because a single operating system would leave the business vulnerable to viral invasions. The conditional reasoning that underlies this argument can be summarized in the following way:

  • Minor Variations → Unauthorized Access Virtually Eliminated

Answer choice (A): This answer choice may be attractive, but it is too strong and therefore incorrect. While it is possible that a business's failure to introduce variety into its operating systems would lead to loss of data, such an outcome is by no means guaranteed. This difference is seen in the stimulus with the phrase "the vandal can then destroy much of the data on all the computers" vs the answer choice, which states that, "it will lose data on its computers ." We cannot be sure the business will for certain lose data, and hence this answer is not an inference.

Answer choice (B): According to the stimulus, a virus can destroy "much of the data on all the computers," not "all the data on [a given] computer." It is imperative to closely watch the words used in the stimulus to describe quantities.

Answer choice (C): This is the correct answer choice. Answer choice (C) is a reiteration of the fourth sentence in the stimulus: if minor variations are introduced, unauthorized access to all computers will be virtually eliminated. The term "virtually" leaves open the possibility that not every single computer will be completely protected from viral invasion.

Answer choice (D): This statement is inconsistent with the argument, which is only about multiple, linked computer systems. Furthermore, it is plausible that computer systems that are not linked can be more easily protected from computer viruses than multiple, linked computer systems. This answer choice is incorrect.

Answer choice (E): While variations can be introduced without any loss of computer compatibility to the business, the author never said that it would be easier to access the data on the computers that use such software. If anything, access to data may become more difficult to protect the it from computer vandalism.
 reop6780
  • Posts: 265
  • Joined: Jul 27, 2013
|
#10167
I could get rid of the rest with certain reasons, but I am not fully convinced of C either.
It is due to the clause after "but will not have protected every computer from viral invasion." Again, owing to the characteristic of Must Be True that I learned from LR bible, I become very cautious of any information that I regard as extraneous.
I cannot find information stating to what extent this variety would prevent computer from virus.

Was it necessary to state "but ~" in C to justify the answer?

Or, was it allowed to be written in such way?

Finally, is "but~" stated based upon the context?

Thank you

Hyun Kim
User avatar
 Dave Killoran
PowerScore Staff
  • PowerScore Staff
  • Posts: 5853
  • Joined: Mar 25, 2011
|
#10190
Hi Hyun,

The question with so-called extraneous information isn't just whether it is extraneous, but whether is is extraneous and also not supported. The test makers have the option of throwing additional, "unneeded" pieces of information into a correct answer choice, as long as that "extra" info is valid. That's what happens here.

Let's look at your questions about this problem, re-ordered:

  • 1. Was it necessary to state "but ~" in C to justify the answer?

    No. They could have left this part off and the answer still would have been correct. So why did they include it? Aside from the fact that it's supported by the stimulus, they did it to throw people off. That's their job :-D

    2. Or, was it allowed to be written in such way?

    Not sure what you mean here. It's in the correct answer, so it is clearly allowed to be done this way :lol:

    3. Finally, is "but~" stated based upon the context?

    Most definitely. when you look at a correct answer and see elements you aren't sure about, go back and try to figure out why it is correct. This is the benefit of practicing and studying--you learn what the test makers are thinking and what they can and cannot do. Based on the statements in the stimulus about computer access, this portion makes sense.

    4. I cannot find information stating to what extent this variety would prevent computer from virus.

    You don't need an exact percent because the answer doesn't trade on exact percents. You just need to know that variety would make it harder for viruses, and that increases protection. Once you have some increase, then you have justification for answer choice (C).

The key in all of the Must questions you are asking about is to look back at the stimulus and try to piece together what the test makers are thinking. They are always right, so it is important to think of each question from their perspective.

Also, be careful with your interpretation of extraneous. I get the sense that you have interpreted those comments to mean that if it wasn't stated exactly in the passage, then the answer is wrong. But that's not the case. You can have answers that draw conclusions (very often, in fact) and you can have answers that include ideas under the "umbrella" of the concepts being discussed (for example, they talk about cats in the stimulus and then one of the answers references "some animals"). As you practice, you'll begin to get a defined sense of how "far" they can go (or will go), and when an answer goes too far. This is one of the immense benefits of looking at as many LSAT questions as possible. But, the first step is to always look at their answers, and ask yourself, "why?" Why is it right or wrong, and what elements do you see as leading to that conclusion.

Thanks and good luck!
 reop6780
  • Posts: 265
  • Joined: Jul 27, 2013
|
#10238
Hi Dave !

I'm truly impressed by your response. Only if I had your explanation on MBT first, I would not have struggled with similar problems for days.

Thank you so much for the details which made me understand MBT better!

Hyun Kim
 ovibalaj
  • Posts: 8
  • Joined: Jun 22, 2015
|
#18985
"Logical Reasoning - Question Type Training" Chapter 2: Must Be True; Question 4; page 21.

Analysis [Primary Objectives]:

* Primary Objective 1.

but = counter-premise indicator
if = introduces a relationship:

Minor Variations → Complete access to all computers is eliminated

Furthermore = addition-premise indicator
Therefore = conclusion indicator

This stimulus is an ARGUMENT.

* Primary Objective 2.
Conclusion is = how do I tell you this without posting copyrighted material?
My Attempt: the conclusion is everything after the word "therefore".

* Primary Objective 3.
This is a strong argument because the pro's of doing such a thing would overcome the con's. Is this a good reason?

* Primary Objective 4.
N/A

* Primary Objective 5.
N/A

* Primary Objective 6.
N/A

* Primary Objective 7.
N/A

* Primary Objective 8.

Answer Choices: Are the reasons below accurate?

A. LOSER
→ language seems to strong. The fact that no variation is included does not automatically mean that a hacker would target those particular computers and destroy the data. It is only a possibility that the hacker might.

B. LOSER
→ this goes against the authors statement. The phrase "all of…" is exaggerating what was said.

C. CONTENDER
→ this is a summary of the "consequences of the statements presented in the stimulus". This answer passes the Fact Test and as such is the correct one.

D. LOSER
→ the problem they address in the stimulus seem to arise from linked computer systems. This answer is backwards.

E. LOSER
→ completely goes against what the author is saying. Logic is reversed.

* Primary Objective 9.

N/A


Thank you again for doing all this for me. I am learning a lot.
 Nikki Siclunov
PowerScore Staff
  • PowerScore Staff
  • Posts: 1362
  • Joined: Aug 02, 2011
|
#18992
Hi Ovi,

Your analysis of the stimulus and the answer choices is absolutely correct. Yes, this is an argument ("therefore..."), and the author presents a strong case for adopting the course of action recommended. (Ultimately, whether this is a strong or a weak argument is irrelevant as we're dealing with a MBT question.)

A few words of advice re: your execution of the Primary Objectives. While diligence is always a good thing, don't let it slow you down: any significant improvement in LR will require you to do 000's of questions, and you can't possibly break them all down to such level of detail. This is fine to do now, while you're still mastering the technique, but I want you to think of the Primary Objectives the way you think about, say, your morning ritual: you wake up, turn the coffee machine on, take a shower, crack two eggs in the skillet (or whatever). The point is, you don't consciously think about every step of the way: you just do then mechanistically. The same should happen with the Primary Objectives: they should become habit-forming after awhile. Here's how I'd reduce them down to 4 simple steps:
  • 1. Read the stimulus
    1.1. If argument, what's the conclusion?
    1.2. If fact set, move to step 2.

    2. Read and ID the question stem

    3. Prephrase.
    3.1. If the stem is in the Help/Hurt families, return to the argument and figure out what's wrong with the conclusion. Then prephrase an answer as per the stem.
    3.2. If the stem is in the Prove family, put the facts together and come up with a conclusion on your own. Use this as your prephrase.

    4. Read the answer choices.
    4.1. Isolate the contenders from the losers. Know exactly why the losers are wrong.
    4.2. If multiple contenders remain, compare them using any applicable techniques (Prove Test, Assumption Negation Technique, Justify Formula, etc.).
Hope this helps a bit! :)
 ovibalaj
  • Posts: 8
  • Joined: Jun 22, 2015
|
#18993
This really helps! Thank you so much! :-D
User avatar
 Dave Killoran
PowerScore Staff
  • PowerScore Staff
  • Posts: 5853
  • Joined: Mar 25, 2011
|
#19017
Hi Ovibalaj,

A quick note about posting copyrighted materials, and how we handle that. Since the questions are owned by LSAC (the makers of the LSAT), they are the ones who object to it being posted. We just follow the rules they have on that :-D

There are a few easy ways to reference parts of the question without violating their rules:

  • The first is to reference line numbers, as in "I thought the conclusion was the last two lines of the stimulus." That's easy for us to follow, and gets the job done in most cases.

    The second is to quote the first and last words of the segment you are referencing. For example: I thought the conclusion was the following, "Some corporations...cannot avoid taxes." That again makes it easy for anyone to follow what you are referencing.

    Last, you can quote very short sections of the question or answer choices without creating a violation. Fair Use laws in the US allow for limited use of copyrighted materials, and with LSAT questions that would typically allow for quoting a phrase here and there. For example: I had a problem understanding this part of the stimulus: "he postulated no identifiable force." It can't be a lengthy part of the problem that is quoted, but this allows you to quote a few words and not create a violation.
If you want to reference the whole question, just identify it by test, section, and number (and maybe a topic identifier so we know we are on the same page), and if you want to reference an answer choice, just use the letter.

Hopefully that will make it easier for you to reference LSAT questions and at the same time now have to worry about violating copyright laws :-D

Thanks!
User avatar
 ccanno
  • Posts: 4
  • Joined: Mar 04, 2021
|
#85190
Hi,

After reading the explanations I understand why "C" is the correct answer. However, I'm still stuck on why "A" is not a good answer.

Thanks,
Christy
 Adam Tyson
PowerScore Staff
  • PowerScore Staff
  • Posts: 5153
  • Joined: Apr 14, 2011
|
#85210
The biggest problem with answer A, Christy, is that it is too certain to be a good inference based on these facts. These businesses "will" lose data is a very strong claim! The stimulus only supports a much softer claim, that they "could" lose data.

A good answer to a Must Be True question should be no more certain or extreme than the facts in the stimulus will support. We know that a computer vandal "can" gain access and destroy the data, but we don't know that they will do so in every case, so we can only select an answer that is about what is possible in this regard.

Pay close attention to the strength of the evidence, and use that to help eliminate answers that are too strong!

Get the most out of your LSAT Prep Plus subscription.

Analyze and track your performance with our Testing and Analytics Package.